Angular Momentum and velocity problem

In summary, the conversation discusses the conservation of linear and angular momentum in a system consisting of a rod and a particle. The rod is struck by the particle at its center and the two objects stick together. The question is asked about the velocity of the end of the rod after the impact. Using conservation of linear momentum, the velocity of the end is found to be half of the initial velocity of the particle. However, when using conservation of angular momentum, the incorrect assumption is made that the rod pivots about one end, leading to an incorrect calculation of the final angular velocity. The correct approach is to consider the angular momentum of the system about its center of mass, which is zero.
  • #1
konichiwa2x
81
0
Hi,

A rod of mass 'm' and length 'l' is lying on a smooth horizontal surface. A particle of mass 'm' is moving with a velocity [tex]v_{0}[/tex] strikes the rod at the centre and sticks to it. What is the velocity of end A after impact? [Solution: [tex]\displaystyle \frac{v_{0}}{2}[/tex]]

http://img131.imageshack.us/img131/2803/angmom1si.png
[/URL]

Here is what I did:

Considering both the particle and the rod as the system, no external torque acts. Therefore, angular momentum can be conserved about any point.
Conserving angular momentum about A,

[tex]\displaystyle mv_{0}\frac{l}{2} = I_{A}\omega[/tex]

[tex]\displaystyle mv_{0}\frac{l}{2} = (\frac{ml^2}{3} + {\frac{ml}{4}}^2)\omega[/tex]

[tex]\displaystyle \omega = \frac{6v_{0}}{7l}[/tex]

Velocity of end A = [tex]\displaystyle \frac{\omega}{(\frac{l}{2})} = \frac{12v_{0}}{7l^2}[/tex]

What am I doing wrong?
 
Last edited by a moderator:
Physics news on Phys.org
  • #2
Hints: Conservation of linear momentum gives you one part. Conservation of angular momentum gives you the other part. What was the total angular momentum of the system before impact? What was the total linear momentum before impact?
 
  • #3
I have done exactly that. The linear momentum before impact is [tex]mv_{0}[/tex] and after impact is [tex]2mv[/tex] where [tex] v = v_{0}/2[/tex]. I did conserve angular momentum (refer my original post). Is there anything wrong in what I did? please help.
 
  • #4
konichiwa2x said:
Is there anything wrong in what I did?
You incorrectly assumed that the rod pivots about one end. It doesn't. The angular momentum about the center of mass of the system remains zero. (And the angular momentum about point A remains [itex]mv_{0}l/2[/itex].)
 
  • #5
Doc Al said:
You incorrectly assumed that the rod pivots about one end. It doesn't. The angular momentum about the center of mass of the system remains zero. (And the angular momentum about point A remains [itex]mv_{0}l/2[/itex].)
This one is confusing me. Using conservation of linear momentum, and noting that the strike is at the cm of the rod, seems to clearly imply no rotation, and thus, the system, ..all parts of it.. moves forward at V_o/2.
But when using conservation of angular momentum, I have the same question as the poster, I think: Why is it acceptable to calculate the initial angular momentum of the particle ((mV_o(r)) about some ficticious point A, about which there is no fixed pivot point, but when calculating the final angular momentum, it is not OK to calculate the angular momentum of the rod about that point, which leads to an erroneus result for omega? (and which, BTW, was incorrectly calculated using an incorrect formula...using v = wr, not v=w/r, yields v = 3V_o/8 at the c.m, which is still the wrong(?) answer ).
 
  • #6
There is no angular momentum before the impact, and none after the impact with the center of mass of the rod. It's a more interesting question when the mass hits one end of the rod and sticks -- I'm having a harder time visualizing the zero angular momentum aspect of that at the moment...
 
  • #7
PhanthomJay said:
Using conservation of linear momentum, and noting that the strike is at the cm of the rod, seems to clearly imply no rotation, and thus, the system, ..all parts of it.. moves forward at V_o/2.
Realize that in noting the implication of "the strike is at the cm of the rod" you are implicitly applying conservation of angular momentum.

But when using conservation of angular momentum, I have the same question as the poster, I think: Why is it acceptable to calculate the initial angular momentum of the particle ((mV_o(r)) about some ficticious point A, about which there is no fixed pivot point, but when calculating the final angular momentum, it is not OK to calculate the angular momentum of the rod about that point, which leads to an erroneus result for omega?
The erroneous result is not due to using point A as a reference point; it's due to assuming that the rod pivots about one end. In general, the angular momentum of an object about some point A is the sum of:
(a) the object's angular momentum about its center of mass
(b) the angular momentum of the object's center of mass about point A​
(and which, BTW, was incorrectly calculated using an incorrect formula...using v = wr, not v=w/r, yields v = 3V_o/8 at the c.m, which is still the wrong(?) answer ).
Again, I think you are assuming the rod pivots about one end.

berkeman said:
It's a more interesting question when the mass hits one end of the rod and sticks -- I'm having a harder time visualizing the zero angular momentum aspect of that at the moment...
In that case, the angular momentum of the system about its center of mass is not zero. The angular momentum about point A is zero, but that just means that the two terms indentified above are equal and opposite.
 

1. What is angular momentum?

Angular momentum is a measure of the rotational motion of an object around a fixed point. It is a vector quantity that depends on the mass, velocity, and distribution of an object's mass around the axis of rotation.

2. How is angular momentum calculated?

Angular momentum is calculated by multiplying the moment of inertia (a measure of how an object's mass is distributed around an axis of rotation) by the angular velocity (the rate of change of the angle of rotation).

3. What is the relationship between angular momentum and velocity?

Angular momentum and velocity are directly proportional to each other. This means that an increase in angular velocity will result in an increase in angular momentum, and vice versa.

4. Can angular momentum be conserved?

Yes, angular momentum can be conserved in a closed system where there are no external torques acting on the system. This is known as the law of conservation of angular momentum.

5. How does angular momentum affect an object's stability?

The greater the angular momentum of an object, the more stable it will be. This is because a higher angular momentum means a greater resistance to changes in its rotational motion, making it less likely to tip over or change direction abruptly.

Similar threads

  • Introductory Physics Homework Help
Replies
3
Views
221
  • Introductory Physics Homework Help
Replies
10
Views
904
  • Introductory Physics Homework Help
10
Replies
335
Views
8K
  • Introductory Physics Homework Help
Replies
11
Views
1K
  • Introductory Physics Homework Help
Replies
10
Views
861
  • Introductory Physics Homework Help
Replies
14
Views
1K
  • Introductory Physics Homework Help
Replies
3
Views
827
  • Introductory Physics Homework Help
Replies
1
Views
1K
Replies
13
Views
898
  • Introductory Physics Homework Help
Replies
5
Views
1K
Back
Top